Line 72: Line 72:
 
\end{pmatrix}</math>  
 
\end{pmatrix}</math>  
  
<math>\left [ A|b \right ] =\begin{bmatrix}  
+
&nbsp; &nbsp; &nbsp; &nbsp;&nbsp;<math>\left [ A|b \right ] =\begin{bmatrix}  
 
1 & 2 & -1 & 5 \\  
 
1 & 2 & -1 & 5 \\  
 
2 & 3 & -1 & 6  
 
2 & 3 & -1 & 6  
Line 80: Line 80:
 
\end{bmatrix}</math>  
 
\end{bmatrix}</math>  
  
<math>x^{\left( 1 \right)}= \begin{bmatrix}
+
&nbsp; &nbsp; &nbsp; &nbsp;&nbsp;<math>x^{\left( 1 \right)}= \begin{bmatrix}
 
-3 & 4 & 0
 
-3 & 4 & 0
 
\end{bmatrix}^{T} \text{ is BFS. } f_{1}=-9</math>  
 
\end{bmatrix}^{T} \text{ is BFS. } f_{1}=-9</math>  
Line 89: Line 89:
 
\end{pmatrix}</math>  
 
\end{pmatrix}</math>  
  
<math>\left [ A|b \right ] =\begin{bmatrix}  
+
&nbsp; &nbsp; &nbsp; &nbsp;&nbsp;<math>\left [ A|b \right ] =\begin{bmatrix}  
 
1 & 2 & -1 & 5 \\  
 
1 & 2 & -1 & 5 \\  
 
2 & 3 & -1 & 6  
 
2 & 3 & -1 & 6  
Line 97: Line 97:
 
\end{bmatrix}</math>  
 
\end{bmatrix}</math>  
  
<math>x^{\left( 2 \right)}= \begin{bmatrix}
+
&nbsp; &nbsp; &nbsp; &nbsp;&nbsp;<math>x^{\left( 2 \right)}= \begin{bmatrix}
 
0 & 1 & -3
 
0 & 1 & -3
 
\end{bmatrix}^{T} \text{ is BFS. } f_{2}=-15</math>  
 
\end{bmatrix}^{T} \text{ is BFS. } f_{2}=-15</math>  
Line 106: Line 106:
 
\end{pmatrix}</math>  
 
\end{pmatrix}</math>  
  
<math>\left [ A|b \right ] =\begin{bmatrix}  
+
&nbsp; &nbsp; &nbsp; &nbsp;&nbsp;<math>\left [ A|b \right ] =\begin{bmatrix}  
 
1 & 2 & -1 & 5 \\  
 
1 & 2 & -1 & 5 \\  
 
2 & 3 & -1 & 6  
 
2 & 3 & -1 & 6  
Line 114: Line 114:
 
\end{bmatrix}</math>  
 
\end{bmatrix}</math>  
  
<math>x^{\left( 2 \right)}= \begin{bmatrix}
+
&nbsp; &nbsp; &nbsp; &nbsp;&nbsp;<math>x^{\left( 2 \right)}= \begin{bmatrix}
 
1 & 0 & -5
 
1 & 0 & -5
 
\end{bmatrix}^{T} \text{ is BFS. } f_{3}=-21</math>  
 
\end{bmatrix}^{T} \text{ is BFS. } f_{3}=-21</math>  
Line 138: Line 138:
 
<math>\color{blue}\text{Solution}</math>  
 
<math>\color{blue}\text{Solution}</math>  
  
<span class="texhtml">The equality constraints can be represented in the form ''A''''x'' = ''b'',</span><br>  
+
<span class="texhtml">The equality constraints can be represented in the form ''A''''x'''''<b> = ''b'',</b></span>'''<br> '''
  
 
<math>A= \begin{bmatrix}
 
<math>A= \begin{bmatrix}
Line 152: Line 152:
 
\end{bmatrix}</math>  
 
\end{bmatrix}</math>  
  
<math>\text{The fist basis candidate is } \begin{pmatrix}
+
<math>\text{The first basis candidate is } \begin{pmatrix}
 
a_{1} &  
 
a_{1} &  
 
a_{2}  
 
a_{2}  
\end{pmatrix}</math><br>
+
\end{pmatrix}</math><br>  
  
<math>\text{The corresponding basic solution is } x^{\left( 1 \right)}= \begin{bmatrix}
+
&nbsp; &nbsp; &nbsp; &nbsp;&nbsp;<math>\text{The corresponding basic solution is } x^{\left( 1 \right)}= \begin{bmatrix}
 
4 & 2 & 0
 
4 & 2 & 0
\end{bmatrix}^{T} \text{ is BFS. The objective function} f_{1}=14</math><br>
+
\end{bmatrix}^{T} \text{ is BFS.}</math><br>  
  
<math>\text{The fist basis candidate is } \begin{pmatrix}
+
&nbsp; &nbsp; &nbsp; &nbsp;&nbsp;<math>\text{The objective function value is } f_{1}=14</math>
 +
 
 +
<math>\text{The second basis candidate is } \begin{pmatrix}
 
a_{1} &  
 
a_{1} &  
 
a_{3}  
 
a_{3}  
 
\end{pmatrix}</math><br>  
 
\end{pmatrix}</math><br>  
  
<math>\text{The fist basis candidate is } \begin{pmatrix}
+
&nbsp; &nbsp; &nbsp; &nbsp;&nbsp;<math>\text{The corresponding basic solution is } x^{\left( 1 \right)}= \begin{bmatrix}
 +
6 & 0 & -2
 +
\end{bmatrix}^{T} \text{, which is not a basic feasible solution.}</math>&nbsp; &nbsp; &nbsp; &nbsp;&nbsp;
 +
 
 +
<math>\text{The third basis candidate is } \begin{pmatrix}
 
a_{1} &  
 
a_{1} &  
 
a_{3}  
 
a_{3}  
\end{pmatrix}</math>
+
\end{pmatrix}</math>
 +
 
 +
&nbsp; &nbsp; &nbsp; &nbsp;&nbsp;<math>\text{The corresponding basic solution is } x^{\left( 1 \right)}= \begin{bmatrix}
 +
0 & 6 & 4
 +
\end{bmatrix}^{T} \text{, which is a basic feasible solution.}</math>
 +
 
 +
&nbsp; &nbsp; &nbsp; &nbsp;&nbsp;<math>\text{The objective function value is } f_{3}=10</math>
 +
 
 +
<math>\Therefore \text{, the optimal solution is }  x^{\left( * \right)}= \begin{bmatrix}
 +
0 & 6 & 4
 +
\end{bmatrix}^{T}</math>
  
<br>
 
  
<br>
 
  
 
----
 
----

Revision as of 21:29, 28 June 2012

ECE Ph.D. Qualifying Exam in "Automatic Control" (AC)

Question 3, Part 3, August 2011

Part 1,2,3,4,5

 $ \color{blue}\text{3. } \left( \text{20 pts} \right) \text{ Solve the following linear program, } $

maximize    − x1 − 3x2 + 4x3

subject to    x1 + 2x2x3 = 5

                   2x1 + 3x2x3 = 6

                   $ x_{1} \text{ free, } x_{2}\geq0, x_{3}\leq0. $


The Fundamental Theorem of Linear Programming that one of the basic feasible solutions is an optimal solution. So we generate all possible basic feasible solutions and select from them the optimal one.


$ \color{blue}\text{Solution 1:} $

$ \left.\begin{matrix} x_{1}+2x_{2}-x_{3}=5 \\ 2x_{1}+3x_{2}-x_{3}=6 \end{matrix}\right\}\Rightarrow x_{1}=5-2x_{2}+x_{3}=3-\frac{3}{2}x_{2}+\frac{1}{2}x_{3} $

$ \Rightarrow x_{2}-x_{3}=4 $

$ \text{It is equivalent to min } x_{1}+3x_{2}-4x_{3} =5-2x_{2}+x_{3}+3x_{2}-4x_{3} = x_{2}-3x_{3}+5, x_{2}\geq0, x_{3}\leq0 $

$ x_{2}-3x_{3}+5 = x_{2}-x_{3}-2x_{3}+5=9-2x_{3}\geq9 $   $ \color{green} \text{constrain: } x_{3}\leq0 \Rightarrow x_{3}=0 $

$ \text{Equivalently, } -x_{1}-3x_{2}+4x_{3}\leq-9 $

$ \text{Equality is satisfied when } x_{3}=0, x_{2} =4+0=4, x_{1}=5-2\times4=-3 $

$ \Rightarrow \left\{\begin{matrix} x_{1}=-3\\ x_{2}=4\\ x_{3}=0 \end{matrix}\right. $

$ \color{green} \text{This solution is not using the Fundamental Theorem of Linear Programming} $


$ \color{blue}\text{Solution 2:} $

One of the basic feasible solution is an optimal solution.
The equality constraints can be represented in the form Ax = b,

$ A=\begin{bmatrix} 1 & 2 & -1\\ 2 & 3 & -1 \end{bmatrix}=\begin{bmatrix} a_{1}& a_{2}& a_{3} \end{bmatrix}; b=\begin{bmatrix} 5\\ 6 \end{bmatrix} $

$ \text{The first basis candidate is } \begin{pmatrix} a_{1} & a_{2} \end{pmatrix} $

        $ \left [ A|b \right ] =\begin{bmatrix} 1 & 2 & -1 & 5 \\ 2 & 3 & -1 & 6 \end{bmatrix} = \cdots = \begin{bmatrix} 1 & 0 & 1 & -3 \\ 0 & 1 & -1 & 4 \end{bmatrix} $

        $ x^{\left( 1 \right)}= \begin{bmatrix} -3 & 4 & 0 \end{bmatrix}^{T} \text{ is BFS. } f_{1}=-9 $

$ \text{The second basis candidate is } \begin{pmatrix} a_{2} & a_{3} \end{pmatrix} $

        $ \left [ A|b \right ] =\begin{bmatrix} 1 & 2 & -1 & 5 \\ 2 & 3 & -1 & 6 \end{bmatrix} = \cdots = \begin{bmatrix} 1 & 0 & 1 & -3 \\ 1 & 1 & 0 & 1 \end{bmatrix} $

        $ x^{\left( 2 \right)}= \begin{bmatrix} 0 & 1 & -3 \end{bmatrix}^{T} \text{ is BFS. } f_{2}=-15 $

$ \text{The third basis candidate is } \begin{pmatrix} a_{1} & a_{3} \end{pmatrix} $

        $ \left [ A|b \right ] =\begin{bmatrix} 1 & 2 & -1 & 5 \\ 2 & 3 & -1 & 6 \end{bmatrix} = \cdots = \begin{bmatrix} 0 & 1 & 1 & 4 \\ 1 & 1 & 0 & 1 \end{bmatrix} $

        $ x^{\left( 2 \right)}= \begin{bmatrix} 1 & 0 & -5 \end{bmatrix}^{T} \text{ is BFS. } f_{3}=-21 $

$ \because f_{1}>f_{2}>f_{3} $

$ \therefore \text{The optimal solution is } x^{*}=\begin{bmatrix} -3 & 4 & 0 \end{bmatrix} \text{ with objective value } -9 $


$ \color{blue} \text{Related Problem: Solve following linear programming problem,} $

minimize   3x1 + x2 + x3

subject to  x1 + x3 = 4

                  x2x3 = 2

                  $ x_{1}\geq0,x_{2}\geq0,x_{3}\geq0, $

$ \color{blue}\text{Solution} $

The equality constraints can be represented in the form A'x = b,

$ A= \begin{bmatrix} 1 & 0 & 1\\ 0 & 1 & -1 \end{bmatrix}=\begin{bmatrix} a_{1}& a_{2}& a_{3} \end{bmatrix}; b=\begin{bmatrix} 4\\ 2 \end{bmatrix} $

$ \text{The first basis candidate is } \begin{pmatrix} a_{1} & a_{2} \end{pmatrix} $

        $ \text{The corresponding basic solution is } x^{\left( 1 \right)}= \begin{bmatrix} 4 & 2 & 0 \end{bmatrix}^{T} \text{ is BFS.} $

        $ \text{The objective function value is } f_{1}=14 $

$ \text{The second basis candidate is } \begin{pmatrix} a_{1} & a_{3} \end{pmatrix} $

        $ \text{The corresponding basic solution is } x^{\left( 1 \right)}= \begin{bmatrix} 6 & 0 & -2 \end{bmatrix}^{T} \text{, which is not a basic feasible solution.} $        

$ \text{The third basis candidate is } \begin{pmatrix} a_{1} & a_{3} \end{pmatrix} $

        $ \text{The corresponding basic solution is } x^{\left( 1 \right)}= \begin{bmatrix} 0 & 6 & 4 \end{bmatrix}^{T} \text{, which is a basic feasible solution.} $

        $ \text{The objective function value is } f_{3}=10 $

$ \Therefore \text{, the optimal solution is } x^{\left( * \right)}= \begin{bmatrix} 0 & 6 & 4 \end{bmatrix}^{T} $



Automatic Control (AC)- Question 3, August 2011

Go to



Back to ECE Qualifying Exams (QE) page

Alumni Liaison

EISL lab graduate

Mu Qiao